2021秋 数分B1笔记

实数

〇、杂项

最小数原理:集合 \(\mathbb{T} \sub \mathbb{N},\mathbb{T} \neq \varnothing\) ,那么 \(\mathbb{T}\) 中有最小数。

证明:

构造集合 \(\mathbb{S} = \{s\mid\forall t \in \mathbb{T}, s \leq t\}\)

显然 $ 1 \in \mathbb{S} \Rightarrow \mathbb{S} \neq \varnothing$

又有 \(\forall t \in \mathbb{T}, t + 1 \notin \mathbb{S} \Rightarrow \mathbb{S} \neq \mathbb{N}\)

所以一定 \(\exist s_0 \in \mathbb{S}, \ s_0 + 1 \notin \mathbb{S}\) (反证法,根据归纳公理显然)

下证 \(s_0 \in \mathbb{T}\) ,考虑反证

\(s_0 \notin \mathbb{T}\),又 \(s_0 \in \mathbb{S}\),所以有 \(\forall t \in \mathbb{T}, s_0 < t\),则有 \(\forall t \in \mathbb{T},s_0 + 1 \leq t \Rightarrow s_0 + 1 \in \mathbb{S}\) ,矛盾

\(s_0 \in \mathbb{T}\)\(\forall t \in \mathbb{T}, s_0 \leq t\)\(s_0\) 则为 \(\mathbb{T}\) 中的最小数,最小数原理得证

一、域的定义

\(\mathbb{F}\) 是集合,具有

  • 加法 \(\forall x, y \in \mathbb{F}, x + y \in \mathbb{F}\)
  • 乘法 \(\forall x, y \in \mathbb{F}, x \cdot y \in \mathbb{F}\)
  • \(0\)\(\forall x \in \mathbb{F},\exist 0 \in \mathbb{F}, 0 + x \in x\)
  • 单位元 \(\exists 1 \in \mathbb{F}, 1 \cdot x = x \cdot 1 = x\)
  • 负元 \(\forall x \in \mathbb{F}, \exist -x \in \mathbb{F}, x + (-x) = 0\)
  • 满足交换律,结合律,交换律

则称 \(\mathbb{F}\) 为一个域

二、有序域

\(\mathbb{F}\) 是一个域,若满足 \(\forall x, y \in \mathbb{F}, x < y, x > y, x = y\) 有且仅有一种成立,则 \(\mathbb{F}\) 有序,称为有序域

复数域 \(\mathbb{C}\) 不是有序域,复数的乘法运算与有序的定义不兼容

三、有界的定义

\(\mathbb{E} \sub \mathbb{F}\) ,若 \(\exists \beta \in \mathbb{F},\forall \alpha \in \mathbb{E}\) ,有 \(\alpha \leqslant(\geqslant) \beta\),则称 \(\beta\)\(\mathbb{E}\) 的上(下)界

四、确界的定义

\(\mathbb{E} \sub \mathbb{F}\) 有上界,若在 \(\mathbb{F}\)\(\mathbb{E}\) 有最小(大)的上(下)界,则称为上(下)确界,记为 \(\sup \mathbb{E} \in \mathbb{F}\) 或者 \(\inf \mathbb{E} \in \mathbb{F}\)

五、确界原理

\(\mathbb{F}\) 中任意有上(下)界子集 \(\mathbb{E}\)\(\mathbb{F}\) 中一定有上(下)确界,则称 \(\mathbb{F}\) 满足上(下)确界原理

定理:若 \(\mathbb{F}\) 满足上确界原理,则 \(\mathbb{F}\) 满足下确界原理

证明:

\(\mathbb{F}\) 满足上确界原理,要证 \(\forall \mathbb{E} \sub \mathbb{F}\)\(\mathbb{E}\) 有下界则一定有下确界

构造 \(\mathbb{E}^\prime = \{\beta\mid\beta\)\(\mathbb{E}\) 的下界\(\} \sub \mathbb{F}\)

定理:存在满足确界原理的有序域

且以 \(\mathbb{Q}\) 为其子集的有序域记为 \(\mathbb{R}\) ,称为实数域

构造性证明 \(\text{Dedekind}\) 分割(摘自知乎,侵删):

假设我们只知道有理数。我们把所有有理数按如下要求装入集合 \(A\)\(A'\)

  • 任一有理数必属于 \(A, A'\) 之一;
  • \(A'\) 中的每一个有理数都大于 \(A\) 中的有理数。

这样操作的对有理数全集的分划 \(A|A'\) 就称为 \(\text{Detekind}\) 分割。

显然在此划分下就会出现三种情况:

  • \(A\) 中有最大数,\(A'\) 中无最小数;
  • \(A\) 中无最大数,\(A'\) 中有最小数;
  • \(A\) 中无最大数,\(A'\) 中也无最小数。

前两种情况属于存在“界数”的情况,为了明确起见,我们约定,若某个分割存在“界数”,就总把这个“界数”放在 \(A'\) 中,于是可以将这两种情况归并为一种;至于第三种情况,则属于不存在“界数”的情况。

如此,每一个 \(\text{Detekind}\) 分割都唯一地定义了一个实数:有界数的,就是定义了这个作为界数的有理数;无界数的,则定义了某个不属于有理数的新数,我们称之为无理数。

容易推知,实数就和 \(\text{Detekind}\) 分割形成双射关系,每一个实数对应一个分划,不同的实数对应不同的分划。

六、实数的特点 \(\begin{cases}①与数轴上的点一一对应\\②不可数\end{cases}\)

\(\text{Archimedes}\) 原理:

\(\forall x, y \in \mathbb{R}\),若\(x > 0,y > 0\),则 \(\exist n \in \mathbb{N}\),使得 \(nx > y \geqslant (n - 1)x\)

\(\mathbb{Q}\)\(\mathbb{R}\) 中稠密:

\(\forall x, y \in \mathbb{R}, x < y\),一定 \(\exists z \in \mathbb{Q},\) 使得 \(x < z< y\)

\(\text{Archimedes}\) 原理证明:

\(\mathbb{E} = \{nx\mid n \in \mathbb{N}\} \sub \mathbb{R}\)

(反证)假如对 $\forall n,nx \leqslant y \Rightarrow y $ 是 \(\mathbb{E}\) 上界 \(\Rightarrow \mathbb{E}\) 有上确界

\(\alpha = \sup \mathbb{E} \in \mathbb{R} \Rightarrow \alpha - x < \alpha \Rightarrow \alpha - x\) 不是 \(\mathbb{E}\) 的上界

\(\exists mx \in \mathbb{E}\),使得 \(\alpha - x < mx \Rightarrow \alpha < (m + 1)x \in \mathbb{E}\) 矛盾

所以 \(\exists n_0\) 使得 \(n_0x > y\)

\(\mathbb{S} = \{n \mid nx > y\} \neq \varnothing\)

\(\exists\) 最小的 \(n\)\(nx > y \geqslant (n - 1)x\)

通过 \(\text{Archimedes}\) 原理证明两实数间存在另一有理数:

\(\forall x, y \in \mathbb{R}, x < y \Rightarrow y - x > 0\)

根据 \(\text{Archimedes}\) 原理,\(\exists n\),使得 \(n(y - x) > 1 \Rightarrow ny > 1 + nx\)

再对 \(1\)\(nx\) 应用 \(\text{Archimedes}\) 原理

\(\exists m\) 使得 \(m \cdot 1 > nx \geqslant (m - 1) \cdot 1 \Rightarrow mx < m \leqslant nx + 1 < ny\)

\(\Rightarrow x < \frac{m}{n} < y\)\(\frac{m}{n} \in \mathbb{Q}\)

七、无限小数

\(\forall x > 1, x \in \mathbb{R}\)

根据 \(\text{Archimedes}\) 原理,\(\exists a_0 \in \mathbb{N},a_0 \leqslant x < a_0 + 1\)

同上,\(\exists a_1 \in \mathbb{N}, a_1 \leqslant 10(x - a_0) < a_1 + 1(0 \leqslant a_1 \leqslant 9)\)

\(\Rightarrow a_0 + \frac{a_1}{10} \leqslant x < a_0 + \frac{a_1}{10} + 1\)

以此类推:\(a_0 + \frac{a_1}{10} + \cdots + \frac{a_n}{10^n} \leqslant x < a_0 + \frac{a_1}{10} + \cdots + \frac{a_n}{10^n} + 1\)


数列

一、数列极限

数列 \(a_1, a_2, a_3, \cdots\) 可看成 \(\mathbb{N} \to \mathbb{R}\) 的一个映射。

二、数列极限的定义

\(\{ a_n \}\) 是实数数列,\(a \in \mathbb{R}\)

对于 \(\forall \varepsilon > 0\),若 \(\exists N\),使得 \(\forall n > N\)\(|a_n - a| < \varepsilon\),则称 \(a\)\(\{a_n\}\) 的极限,记作 \(\lim\limits_{n \to \infty} a_n = a\)\(a_n \to a(n \to \infty)\)

\(\exists \varepsilon_0 > 0\),对 \(\forall N\)\(\exists n > N\),但是 \(|a_n - a| \geqslant \varepsilon_0\),那 \(a\) 就不是 \(\{a_n\}\) 的极限

三、性质 \(\begin{cases}① 唯一性,改变有限项不影响敛散性 \\②收敛必有界\ 保序性\\③四则运算(下文略)\\④夹逼定理(下文略)\end{cases}\)

  1. \(\{a_n\}\) 收敛,则 \(\lim\limits_{n \to \infty} a_n\) 唯一。

    证明:

    考虑反证,若有两个极限,分别记为 \(a, b(a < b)\),取 \(\varepsilon_0 = \frac{|a - b|}{2}\)

    根据极限的定义,\(\exists N_1,n > N_1\) 时有 \(|a_n - a| < \varepsilon_0\)\(\exists N_2,n > N_2\) 时有 \(|a_n - b| < \varepsilon_0\)

    \(N = \max\{N_1, N_2\}\),当 \(n > N\)

    \(\begin{aligned}|a - b| & = |a - a_n + a_n - b| \\ & \leqslant |a - a_n| + |a_n - b| \\& < 2\varepsilon\end{aligned}\)

    矛盾

  2. 改变 \(\{a_n\}\) 的有限项,不改变 \(\{a_n\}\) 的敛散性

    不妨设 \(a_n \to a (n \to \infty)\) ,改变有限项的值后,依然 \(\exists n_r\),使得 \(a_n(n \geqslant n_r)\) 不变。

    \(\forall \varepsilon > 0,\exists N > n_r, \forall n > N,|a_n - a| < \varepsilon \Rightarrow \lim\limits_{n \to \infty} a_n = a\),敛散性不变。发散数列同理。

  3. \(\{a_n\}\) 有界,\(\exists M\),使得 \(|a_n| \leqslant M\)\(\forall n\) 成立

  4. \(a > l\),则对充分大 \(n\)\(a_n > l\)

    \(\varepsilon = a - l > 0\)

    \(\exists N,\forall n > N,|a_n - a| < \varepsilon\)

    \(a + \varepsilon > a_n > a - \varepsilon = a - (a - l) = l\)

    得到 \(a_n > l\)

  5. 若对充分大 \(n\)\(a_n \geqslant l\),则 \(a \geqslant l\)

四、子列

定理:\(\{a_n\}\) 收敛 \(\Rightarrow {a_n}\) 的任一子列收敛于同一值(证明略)

推论:若 \(\{a_n\}\) 的某一个子列发散,则 \(\{a_n\}\) 也发散;若存在两个子列收敛于不同值,则 \(\{a_n\}\) 发散

五、实数完备性的若干等价命题

  1. 确界原理:\(\mathbb{R}\) 中任何有上(下)界的子集 \(\mathbb E\) 必有上(下)确界

    定义:若 \(\{a_n\}\) 满足 \(\forall n,a_n \leqslant(\geqslant) a_{n + 1}\),则称 \(\{a_n\}\) 单调递增(减)。不取等的情况称为严格单调递增(减)

    定理:单调递增(减)有上(下)界的数列必收敛

    \(\{a_n\} \uparrow\) 有上界,根据确界原理,\(\{a_n\} \sub \mathbb R\) 必有上确界 \(a = \sup\{a_n\}\)

    \(\Rightarrow \forall \varepsilon > 0,a - \varepsilon\) 不再是上界 \(\Rightarrow \exists a_{n_0} \in \{a_n\}\) 使得 \(a - \varepsilon < a_{n_0}\)

    所以 \(\forall n > n_0,a - \varepsilon < a_{n_0} \leqslant a_n < a < a + \varepsilon\),根据数列极限定义,\(\{a_n\}\) 收敛

    定理:设 \(e_n = \left(1 + \frac{1}{n}\right)^n\),则 \(\{e_n\}\) 收敛

    先证 \(\{e_n\}\) 单调递增:

    \(\begin{aligned}e_{n} &= \left(1 + \frac 1 n\right)^n \\&= 1 + \frac n {1!} \cdot \frac 1 n + \frac {n(n - 1)} {2!} \cdot \frac 1 {n^2} + \frac {n(n - 1)(n - 2)} {3!} \cdot \frac 1 {n^3} + \cdots + \frac {n(n - 1) \cdots 2 \cdot 1} {n!} \cdot \frac 1 {n^n} \\&= 1 + 1 + {1 \over 2!}\left(1 - \frac 1 n\right)+{1 \over 3!}\left(1 - \frac 1 n\right)\left(1 - \frac 2 n\right) + \cdots + {1 \over n!}\left(1 - \frac 1 n\right)\left(1 - \frac 2 n\right) \cdots \left(1 - \frac {n - 1} n\right)\end{aligned}\)

    类似地:

    \(e_{n + 1} = 1 + 1 + {1 \over 2!}\left(1 - \frac 1 {n + 1}\right)+{1 \over 3!}\left(1 - \frac 1 {n + 1}\right)\left(1 - \frac 2 {n + 1}\right) + \cdots + {1 \over (n + 1)!}\left(1 - \frac 1 {n + 1}\right)\left(1 - \frac 2 {n + 1}\right) \cdots \left(1 - \frac n {n + 1}\right)\)

    作差不难发现 \(e_{n + 1} > e_n\)

    又根据

    \(\begin{aligned}e_n & < 1 + 1 + {1 \over 2!} + {1 \over 3!} + \cdots + {1 \over n!} \\&< 1 + 1 + {1 \over 2} + {1 \over 2^2} + \cdots + {1 \over 2^{n-1}} \\&= 1 + {1 - {1 \over 2^n} \over 1 - {1 \over 2}} = 3 - {1 \over 2^{n - 1}} <3\end{aligned}\)

    所以 \(e_n\) 有上界,单调递增有上界必收敛,记为 \(\lim\limits_{n \to \infty}e_n = e\)

  2. 列紧性

    区间套定理:设有一列区间 \([a_n, b_n], n = 1, 2, \cdots\) 满足

    • \([a_{n+1}, b_{n + 1}] \sub [a_n, b_n],n = 1, 2, \cdots\)
    • \(\lim\limits_{n \to \infty}(b_n - a_n) = 0\)

    则有且仅有一点 \(\xi \in [a_n, b_n],n = 1, 2, \cdots\)

    证明:

    \(\{a_n\}\) 递增有上界 \(b_1\)\(\{b_n\}\) 递减有下界 \(a_1\) \(\Rightarrow\) \(\{a_n\},\{b_n\}\) 均收敛

    \(\lim\limits_{n \to \infty} a_n = a,\lim\limits_{n \to \infty} b_n = b\)

    根据数列极限的保序性:\(a_n < b_n \Rightarrow a \leqslant b\)

    \(0 \leqslant b - a \leqslant \lim\limits_{n \to \infty}(b_n - a_n) = 0 \Rightarrow b = a = \xi\)

    容易验证 \(\xi \in [a_n, b_n], \forall n \in \N\)

    推广可得 \(\bigcap\limits_{n = 1}^\infty[a_n, b_n] \neq \varnothing\),特别地若 \(\lim\limits_{n \to \infty}(b_n - a_n) = 0\)\(\bigcap\limits_{n = 1}^\infty[a_n, b_n] = \{\xi\}\)

    \(\text{Bolzano-Weierstrass}\) 定理:任何有界数列必有收敛子列

    \(\{x_n\}\) 是有界数列, 设 \(\{x_n\} \sub [a, b]\),则区间 \([a, \frac{a + b}{2}],[\frac{a + b}{2}, b]\) 中至少有一个包含 \(\{a_n\}\) 中的无限项

    记为 \([a_1, b_1]\),对 \([a_1, b_1]\) 重复上述过程得到 \([a_2, b_2]\),以此类推将得到 \(k\) 个区间,满足

    \([a_1, b_1] \supset [a_2, b_2] \supset \cdots \supset [a_k, b_k]\)

    \(b_k - a_k = {1 \over 2^k}(b - a) \to 0,k \to \infty\)

    根据区间套定理 \(\lim\limits_{k \to \infty} a_k = \lim\limits_{k \to \infty} b_k = x\)

    再根据夹逼定理,\([a_k, b_k]\) 内所有 \(x_i\) 组成的子列收敛于 \(x\)

  3. \(\text{Cauchy}\) 收敛准则

    定义:\(\{a_n\}\) 满足 \(\forall \varepsilon > 0,\exists N\),当 \(n, m > N\) 时有 \(|a_n-a_m | < \varepsilon\) ,或者写成 \(| a_{n+p} - a_n | < \varepsilon\)\(\forall p\) 成立

    ​ 则称 \(\{a_n\}\) 为基本列

    定理:\(\{a_n\}\) 收敛 \(\iff\) \(\{a_n\}\) 是基本列

六、发散到无穷大

定义:

\(\lim\limits_{n \to \infty} a_n = +\infty,\forall M > 0,\exists N\),当 \(n > N\)

  • \(a_n > M\),则称 \(a_n\) 发散到 \(+\infty\)
  • \(a_n < -M\),则称 \(a_n\) 发散到 \(-\infty\)
  • \(|a_n| > M\),则称 \(a_n\) 发散到 \(\infty\)

定理:单调递增数列发散到 \(+\infty \iff\) 数列无界 \(\forall M > 0,\exists a_N\),使得 \(a_N > M\)

七、\(\text{Stolz}\) 定理

定理(“\(\frac \infty \infty\)”型或“\(\frac x \infty\)” 型):设 \(\{a_n\},\{b_n\}\) 满足

  1. \(b_n\) 严格 \(\uparrow\)\(\lim\limits_{n \to \infty}b_n = +\infty\) (不要求 \(a_n \to +\infty\)
  2. \(\lim\limits_{n \to \infty} {a_{n + 1} - a_n \over b_{n + 1} - b_n} = A\) (可以是无穷)

则有 \(\lim\limits_{n \to \infty}{a_n \over b_n} = A\)

定理(“\(\frac 0 0\)” 型)

  1. \(a_n \to 0, b_n \to 0\),且 \(\{b_n\}\) 严格 \(\downarrow\)
  2. \(\lim\limits_{n \to \infty} {a_{n + 1} - a_n \over b_{n + 1} - b_n} = A\)

则有 \(\lim\limits_{n \to \infty}{a_n \over b_n} = A\)

函数极限

一、函数

映射:\(f:X \to Y\) 单射

函数:\(f:I(\subset \R) \to J(\subset \R)\)

  1. 三种情况:

    1. 有界:\(\forall x \in I,|f(x)| \leqslant M\) 或者 \(m \leqslant f(x) \leqslant M\)
    2. 单调:\(\forall x_1, x_2 \in I(x_1 < x_2) \Rightarrow f(x_1) \leqslant f(x_2)\) 则单调递增,不取等则严格;递减同理
    3. 一一对应:\(\forall x_1 \neq x_2 \Rightarrow f(x_1) \neq f(x_2)\)
    4. 反函数:\(\forall y \in J,\exists\) 唯一 \(x \in I\),使得 \(y = f(x)\),则 \(x = f^{-1}(y)\)
  2. 函数的复合:\(y = g(f(x))\)

  3. 初等函数

    1. 幂函数:\(f(x) = x^\alpha(x > 0,\alpha \in \R)\)

    2. 指数函数:\(f(x) = a^x(a > 0,a \neq 1)\)

    3. 对数函数:\(f(x) = \log_ax(x > 0, a > 0, a \neq 1), f(x) = \ln x\)

    4. (反)三角函数

    5. 双曲函数

      \(\sinh x = {e^x - e^{-x} \over 2},\cosh x = {e^x + e^{-x} \over 2},\cosh^2x - \sinh^2x = 1,\cos x = {e^{ix} + e^{-ix} \over 2}, \sin x = {e^{ix} + e^{-ix} \over 2i}\)

    初等函数通过有限次四则运算和复合运算,仍然为复合函数

  4. 函数的其他表达方式

    1. 分段函数
    2. 隐函数
    3. 参数方程

二、在无穷处的极限

\(\forall \varepsilon > 0,\exists X > 0\),当 \(|x| > X\) 时,有 \(|f(x) - a| < \varepsilon\),则称 \(a\)\(f(x)\) 在无穷处的极限,记为 \(\lim\limits_{x \to \infty}f(x) = a\)

单侧极限 \(\begin{cases}\lim\limits_{x \to +\infty}f(x) = a \\ \lim\limits_{x \to -\infty}f(x) = a\end{cases}\) . 结论:\(\lim\limits_{x \to \infty}f(x) = a \iff \begin{cases}\lim\limits_{x \to +\infty}f(x) 存在 \\ \lim\limits_{x \to -\infty}f(x) 存在\end{cases} 且相等\)

三、在有限点的极限

\(\lim\limits_{x \to x_0}f(x) = a \iff \forall \varepsilon > 0, \exists \delta > 0\),当 \(0 < |x - x_0| < \delta(x \in \mathring{U}(x_0, \delta))\) 时,有 \(|f(x) - a| <\varepsilon\)

结论:\(f(x)\)\(x_0\) 处有极限 \(\iff\) \(f(x)\)\(x_0\) 处的左右极限存在且相等

四、性质与判别法

一、极限若存在则唯一

二、局部有界

  1. \(f(x)\)\(x_0\) 附近有界(\(\exists \delta > 0, x \in \mathring{U}(x_0, \delta)\)

  2. \(\alpha < a < \beta\),则在 \(x_0\) 附近有 \(\alpha \leqslant f(x) \leqslant \beta\)

    \(\varepsilon = \min\{\beta - a, a - \alpha\} > 0, \exists \delta > 0\)\(x \in \mathring{U}(x_0, \delta)\) 时,\(\alpha \leqslant a - \varepsilon < f(x) < a + \varepsilon \leqslant \beta\)

三、四则运算

\(f(x) \to a, g(x) \to b(x \to x_0)\),则

  1. \(\lim\limits_{x \to x_0}(\alpha f(x) + \beta g(x)) = \alpha a + \beta b\)
  2. \(\lim\limits_{x \to x_0}f(x)g(x) = ab\)
  3. \(\lim\limits_{x \to x_0}{f(x) \over g(x)} = \frac a b(b \neq 0)\)

四、保序性

\(x_0\) 附近有 \(f(x) \leqslant g(x) \Rightarrow \lim\limits_{x \to x_0} f(x) \leqslant \lim\limits_{x \to x_0}g(x)\)

五、复合函数

\(f(x)\)\(x_0\) 附近,\(g(t)\)\(t_0\) 附近均有定义,当 \(t \neq t_0,g(t) \neq x_0\)

​ 若 \(\lim\limits_{x \to x_0}f(x) = l, \lim\limits_{t \to t_0} g(t) = x_0\),则 \(\lim\limits_{t \to t_0}f(g(t)) = l\)

证明:

\(\forall \varepsilon > 0, \exists \tau > 0\),当 \(0 < |x - x_0| < \tau\) 时,有 \(|f(x) - l| < \varepsilon\)

\(\tau > 0,\exists \delta > 0\),当 \(0 < |t - t_0| < \delta\) 时,有 \(0 < |g(t) - x_0| < \tau \Rightarrow |f(g(t)) - l| < \varepsilon \Rightarrow \lim\limits_{t \to t_0}f(g(t)) = l\)

证毕

六、函数与数列复合

\(\lim\limits_{x \to x_0}f(x) = a \iff\) 对任意收敛于 \(x_0\) 的数列 \(\{a_n\}(a_n \neq x_0)\),有 \(\lim\limits_{n \to \infty}f(a_n) = a\)

证明:

\((\Rightarrow)\)

对于 \(\forall \{a_n\}, a_n \to x_0(x \to \infty)\) ,要证 \(\{f(a_n)\}\) 收敛于 \(a\)

即证 \(\forall \varepsilon > 0,\exists \delta > 0\) 使得 \(0 < |x - x_0| < \delta\) 时,有 \(|f(x) - a| < \varepsilon\)

\(\delta > 0,\exists N\),当 \(n > N\) 时有 \(0 < |a_n - x_0| < \delta \Rightarrow\)\(n > N\) 时,\(|f(a_n) - a| < \varepsilon\)

\((\Leftarrow)\)

假如当 \(x \to x_0\)\(f(x)\) 不以 \(a\) 为极限

\(\Rightarrow \exists \varepsilon_0 > 0\),使得对 \(\forall \delta > 0\),即使 \(0 < |x - x_0| < \delta\),但是 \(|f(x) - a| \geqslant \varepsilon_0\)

\(\delta = \frac 1 n\),取 \(a_n = x_0 + \frac 1 {2n} \to x_0\)

\(0 < |a_n - x_0| < \frac 1 n = \delta\) 时,但是 \(|f(a_n) - a| \geqslant \varepsilon_0\),矛盾

七、夹逼定理

\(x_0\) 附近有 \(g(x) \leqslant f(x) \leqslant h(x)\),且 \(\lim\limits_{x \to x_0}g(x) = \lim\limits_{x \to x_0}h(x) = a \Rightarrow \lim\limits_{x \to x_0}f(x) = a\)

八、单调性与极限存在的关系

\(f(x)\)\((a, b)\) 上单调,则对 \(\forall x_0 \in (a, b), \lim\limits_{x \to x_0^{\pm}}f(x)\) 存在

证明:

\(f(x) \uparrow\)\(\mathbb E = \{f(x) \mid x < x_0\}\) 有上界

\(\exists\) 上确界 \(l\)\(\forall \varepsilon > 0 \Rightarrow l - \varepsilon\) 不是 \(\mathbb E\) 的上界

\(\exists \overline x < x_0\),但是 \(f(\overline x) > l - \varepsilon\)

\(0 < \delta < x_0 - \overline x\),当 \(0 < x_0 - \overline x < \delta\) 时,有 \(l - \varepsilon < f(\overline x) \leqslant l\)

\(\Rightarrow \lim\limits_{x \to x_0^-}f(x)\) 存在,另一种情况同理,证毕

九、\(\text{Cauchy}\) 收敛准则

\(f(x)\)\(x_0\) 附近有定义

\(f(x)\)\(x_0\) 处有极限 \(\iff\) \(\forall \varepsilon > 0, \exists \delta > 0\),当 \(0 < |x^\prime - x_0| < \delta, 0 < |x^{\prime\prime} - x_0| < \delta\) 时,有 \(|f(x^\prime) - f(x^{\prime\prime})| < \varepsilon\)

证明:

\((\Rightarrow)\)

设极限为 \(a\)\(\forall \varepsilon > 0, \exists \delta > 0\),当 \(0 < |x - x_0| < \delta\) 时,有 \(|f(x) - a| < \frac \varepsilon 2\)

\(0 < |x^\prime - x_0| < \delta, 0 < |x^{\prime\prime} - x_0| < \delta\)

\(|f(x^\prime) - a| < \frac \varepsilon 2, |f(x^{\prime\prime}) - a| < \frac \varepsilon 2 \Rightarrow |f(x^\prime) - x^{\prime\prime}| < \varepsilon\)

\((\Leftarrow)\)

因为对 \(\forall \varepsilon > 0, \exists \delta > 0\),当 \(0 < |x^\prime - x_0| < \delta, 0 < |x^{\prime\prime} - x_0| < \delta\) 时,有 \(|f(x^\prime) - x^{\prime\prime}| < \varepsilon\)

任取 \(a_n \to x_0(a_n \neq x_0) \Rightarrow \exists N\),当 \(n, m > N\) 时, \(0 < |a_n - x_0| < \delta, 0 < |a_m - x_0| < \delta\)

\(\Rightarrow |f(a_n) - f(a_m)| < \varepsilon \Rightarrow \{f(a_n)\}\) 满足 \(\text{Cauchy}\) 收敛准则,设 \(f(a_n) \to l\)

根据函数与数列复合的极限, \(\lim\limits_{x \to x_0}f(x) = l\),极限存在,证毕

五、两个重要极限

\[\lim\limits_{x \to 0}{\sin x \over x} = 1 \]

证明:
待填坑

\[\lim\limits_{x \to \infty}\left(1 + \frac 1 x\right)^x = e \]

证明:

已知 \(\lim\limits_{n \to \infty}\left(1 + \frac 1 n\right)^n = e\),考虑将自然数的情况推广到正实数:

\(\text{Archimedes}\) 原理,\(\exists n \in \N,n \leqslant x < n + 1\)

则有 \(\left(1 + {1 \over n + 1}\right)^n < \left(1 + {1 \over x}\right)^x < \left(1 + {1 \over n}\right)^{n + 1}\),之后根据夹逼不难证明 \(\lim\limits_{x \to +\infty}\left(1 + {1 \over x}\right)^x = e\)

负数的情况类似

六、无穷大与无穷小量

\(\lim\limits_{x \to x_0}f(x) = \infty \iff \forall M > 0, \exists \delta > 0, \forall x \in \mathring{U}(x_0, \delta), |f(x)| > M\)

\(\lim\limits_{x \to x_0}f(x) = 0 \iff \forall \varepsilon > 0, \exists \delta > 0, \forall x \in \mathring{U}(x_0, \delta), |f(x)| < \varepsilon\)

\(f(x) \to \infty, g(x) \to \infty (x \to x_0)\),则 \(\lim\limits_{x \to x_0}{f(x) \over g(x)} = \begin{cases} 0 & f = o(g) \\ C \neq 0 & f \sim g \\ \infty & g = o(f)\end{cases}\)

函数的连续性

一、连续

  1. 在一点连续

    \(f(x)\)\(x_0\) 的邻域内有定义且在 \(x_0\) 处有极限,若 \(\lim\limits_{x \to x_0}f(x) = f(x_0)\),则称 \(f(x)\)\(x_0\) 连续

  2. 单侧连续

    \(\lim\limits_{x \to x_0^\pm}f(x) = f(x_0\pm0) = f(x_0)\),则称单侧连续

  3. 在区间上连续

    \(\forall x_0 \in I\)\(f(x)\)\(x_0\) 处连续

二、间断

第一类间断点 \(\begin{cases}f(x_0 + 0) = f(x_0 - 0) \neq f(x_0) & 可去间断点\\f(x_0+0) \neq f(x_0 - 0) & 跳跃间断点\end{cases}\)

第二类间断点 \(f(x_0\pm0)\) 至少有一个不存在

三、性质

  1. 局部有界

    \(f(x)\)\(x_0\) 连续,则存在 \(x_0\) 的一个邻域,使得 \(f(x)\) 在邻域内有界

  2. 四则运算(文略)

  3. \(f(x)\)\(x_0\) 连续,\(g(t)\)\(t_0\) 连续且 \(g(t_0) = x_0\)\(f(g(t))\)\(t_0\) 处连续

    \[\lim\limits_{t \to t_0}f(g(t)) = f(g(t_0)) = f(\lim\limits_{t \to t_0}g(t)) \]

  4. \(f(x)\)\([a, b]\) 上连续,则(缓证)

    1. \(f(x)\)\([a, b]\) 上有反函数 \(\iff\) \(f\)\([a, b]\) 上严格单调
    2. \(f(x)\) 的反函数 \(f^{-1}(x)\)\([a, b]\) 上也一定连续

四、初等函数连续性

结论:初等函数在其定义域内一定连续

五、闭区间上连续函数的性质

\(f(x)\mid_{[a, b]}\) 连续

介值性

定理(零点定理)

\([a, b]\) 上的连续函数 \(f(x)\),若 \(f(a) \cdot f(b) < 0\),则一定存在 \(\xi \in (a, b), f(\xi) = 0\)

证明:

不妨设 \(f(a) < 0, f(b) > 0\)

  1. \([a_m, b_m] \sub [a_{m - 1}, b_{m - 1}] \sub \cdots \sub [a, b]\)
  2. \(b_n - a_n = \frac 1 2(b - a) \to 0, a_n \to \xi, b_n \to \xi\)

\(\Rightarrow \exists \xi \in [a, b]\),使得 \(a_n \leqslant \xi \leqslant b_n\)

\(f(a_n) < 0 \Rightarrow f(\xi) \leqslant 0,f(b_n) > 0 \Rightarrow f(\xi) \geqslant 0\)

\(\Rightarrow f(\xi) = 0\),证毕

定理(介值定理)

\(\{f(x)\} \sub [a, b], f(a) \neq f(b)\),则对 \(\forall\) 介于 \(f(a), f(b)\) 之间的 \(r\),一定存在 \(f(\xi) = r\)

不动点

\(f([a, b]) = \{f(x) \mid x \in [a, b]\} \sub [a, b]\),则一定 \(\exists x_0 \in [a, b], f(x_0) = x_0\)

有界性

定理:闭区间上连续函数一定有界

证明:

假设存在闭区间 \([a, b]\) 上的某个连续函数无界

\(\forall N, \exists x_n \in [a, b]\),使得 \(|f(x_n)| \geqslant N\) \(\Rightarrow \exists\) 子列 \(x_{n_k} \to x_0 \Rightarrow x_0 \in [a, b]\)(保序性)

又根据 \(f(x)\) 的连续性, \(N_k \leqslant |f(x_{n_k})| \to |f(x_0)|\),有界,矛盾

定理:闭区间上连续函数必取到最大(小)值

证明:

原命题即 \(\exists x^*, x_* \in [a, b]\),使得 \(f(x^*) \geqslant f(x) \geqslant f(x_*)\)\(\forall x \in [a, b]\) 成立

\(f(x)|_{[a, b]}\) 有界因此有上确界,记 \(M = \sup f(x), \forall n \in \N, M - \frac 1 n\) 不是上界

\(\exists x_n \in [a, b]\) 使得 \(M \geqslant f(x_n) \geqslant M - \frac 1 n\)

\(\Rightarrow \exists x_{n_k} \to x^* \in [a, b]\) 再令 \(n \to \infty\) 根据夹逼 \(f(x^*) = M \Rightarrow f(x^*)\)\(f(x)\)\([a, b]\) 的最大值

最小值同理

定理:闭区间上的连续函数的值域也是闭区间 显然

定理: 闭区间上 \(f(x)\) 连续,$f(x) $有反函数 \(\iff\) \(f(x)\)\(f^{-1}(x)\) 连续且严格单调

证明:

\((\Leftarrow)\) 显然

\((\Rightarrow)\) 考虑反证

\(\exists x_1 < x_2 < x_3\),使得 \(f(x_2)\) 不在 \(f(x_1)\)\(f(x_3)\) 之间

不妨设 \(f(x_2) < f(x_1) < f(x_3) \Rightarrow \exists \xi \in (x_2, x_3), f(\xi) = f(x_1)\),矛盾

已知反函数严格单调,下证连续,即证 \(\forall \varepsilon > 0 \exists, \delta > 0\),当 \(|y - y_0| < \delta\) 时,\(|f^{-1}(y) - f^{-1}(y_0)| < \varepsilon\)

\(x_0 = f^{-1}(y_0),y_1 = f(x_0 - \varepsilon), y_2 = f(x_0 + \varepsilon) \Rightarrow y_1 < y_0 < y_2\)

\(\delta = \min\{y_0 - y_1, y_2 - y_0\}\),当 \(|y - y_0| < \delta\)

\(y_1 < y < y_0 + \delta \leqslant y_2 \Rightarrow f^{-1}(y_1) < f^{-1}(y) < f^{-1}(y_2) \Rightarrow |f^{-1}(y) - f^{-1}(y_0)| < \varepsilon\)

六、一致连续性

定义:设 \(f(x)\)\(I\) 上有定义,\(\forall \varepsilon > 0\),如果 \(\exists \delta > 0\),使得对 \(\forall x_0 \in I\)

​ 当 \(|x - x_0| < \delta\) 时,有 \(|f(x) - f(x_0)| < \varepsilon\) 成立,则称 \(f(x)\)\(I\) 上一致连续

等价定义:\(\forall \varepsilon > 0, \exists \delta > 0\),只要 \(|x' - x''| < \delta\),就有 \(|f(x') - f(x'')| < \varepsilon\)

定理:闭区间上连续的函数一定一致连续

证明(反证):

\(\exists \varepsilon_0 > 0\),对 \(\forall n, \delta_n = \frac 1 n\),当 \(|x_n - y_n| < \delta_n\) 时,有 \(|f(x_n) - f(y_n)| \geqslant \varepsilon_0\)

根据子列的收敛性可以证明 \(\lim\limits_{k \to \infty}|f(x_{n_k}) - f(y_{n_k})| = 0\),矛盾

微分

一、导数

定义:设 \(f(x)\)\(I\) 上有定义,\(x_0 \in I\),若 \(\lim\limits_{x \to x_0}{f(x) - f(x_0) \over x - x_0}\) 存在且有限,则称 \(f(x)\)\(x_0\) 处可导,记为

\[f'(x_0) = \lim\limits_{x \to x_0}{f(x) - f(x_0) \over x - x_0} \]

单侧导数

\[\lim\limits_{\Delta x \to 0^\pm}{f(x_0 + \Delta x) - f(x) \over \Delta x} = f'_\pm(x_0) \]

结论:左右导数存在且相等 \(\iff\) 导数存在

二、性质

  1. 可导一定连续(连续不一定可导)

  2. 四则运算

    1. \((f(x) \pm g(x))' = f'(x) \pm g'(x)\)
    2. \((f(x)g(x))' = f'(x)g(x) + f(x)g'(x)\)
    3. \(\left({f(x) \over g(x)}\right)' = {f'(x)g(x) - f(x)g'(x) \over (g(x))^2}(g(x) \neq0)\)
  3. 复合函数求导

    \(y = f(x), x = g(t)\) 都可导,则 \(y = f(g(t))\) 也可导(链式法则证明)

三、高阶导数

\(f|_I\) 可导, \(f'|_I\) 函数若仍然可导,则 \(f''(x)|_I, \cdots\) 称为 \(f(x)\) 的高阶导数,记为 \(f^{(n)}(x) = (f^{n - 1}(x))'\),或记为

\[{\text d^n y \over \text d x^n} = {\text d \over \text d x}\left({\text d^{n - 1} y \over \text d x^{n - 1}}\right) \]

\(\text{Leibnis}\) 定理:

\[(f \cdot g)^{(n)} = \sum\limits_{k = 0}^n{n \choose k}f^{(n - k)} \cdot g^{(k)} \]

\(\exists x_0 \in I, f(x_0) = f'(x_0) = \cdots = f^{(r)}(x_0) = 0\),则称 \(x_0\)\(f(x)\)\(r\) 重零点

常用导数

  1. \((\sin x)^{(n)} = \sin(x + {n\pi \over2})\)

  2. 考虑 \(y = \arctan x\) 的在 \(x = 0\) 处的高阶导数

    一次求导:\(y'(1 + x ^ 2) = 1\)

    对该恒等式求 \(n - 1\) 阶导,根据 \(\text{Leibnis}\) 公式:

    \[(1 + x ^ 2)y^{(n)} + 2(n - 1)xy^{(n - 1)} + (n - 1)(n - 2)y^{(n - 2)} = 0 \]

    \(x = 0\) 代入得到:

    \[y^{(n)}(0) = -(n - 1)(n - 2)y^{(n - 2)}(0) \]

    又有 \(y(0) = 0, y'(0) = 1\),分奇偶分析可得:

    \[y^{(2k + 1)}(0) = (-1)^k(2k)!, y^{(2k)}(0) = 0, k = 1, 2, \cdots. \]

四、参数表示的导数

对于参数方程

\[\begin{cases} x = x(t) \\ y = y(t)\end{cases}, t \in [\alpha, \beta] \]

表示的函数可导,如果 \(x'(t) \neq 0\), 且 \(x = x(t)\) 存在可导的反函数 \(t = t(x)\),则有

\[{\text dy \over \text dx} = {\text dy \over \text dt}{\text dt \over \text dx} = {y'(t) \over x'(t)} \]

类似的可以求得

\[{\text d^2y \over \text dx^2} = {y''(t)x'(t) - y'(t)x''(t) \over (x'(t))^3} \]

五、微分

定义:对于 \(y = f(x)\),若存在一条直线使得 \(f(x) = f(x_0) + A(x - x_0) +o(\Delta x)\),则称 \(f(x)\) 可微

结论:可微

\[\Rightarrow \lim\limits_{\Delta x \to 0}{f(x_0 + \Delta x) - f(x_0) \over \Delta x} = A \]

\(\Rightarrow\) 可导且 \(f'(x_0) = A\)

六、微分中值定理

一、极值

定义:设 \(f(x)\)\(I\) 上有定义,若 \(x_0 \in I\) 满足 \(\exists \delta > 0, |x - x_0| < \delta\)

\(f(x) \leqslant f(x_0)\) 则称 \(f(x_0)\) 是极大值;\(f(x) \geqslant f(x_0)\) 则称 \(f(x_0)\) 是极小值

二、\(\text{Fermat}\) 定理

\(f(x)\)\(I\) 内部一点 \(x_0\) 取极值,且 \(f(x)|_I\) 可导,则 \(f'(x_0) = 0\)(根据导数的定义不难证明)

三、\(\text{Rolle}\) 定理

\(f(x)\)\([a, b]\) 上连续,在 \((a, b)\) 上可导,\(f(a) = f(b)\),则 \(\exists \xi \in (a, b), f'(\xi) = 0\)

证明:连续则有最值,不妨设 \(f(x)\) 不是常值函数 \(\Rightarrow\) 最大(小)值 \(\neq f(a) = f(b)\)

不妨设最大值点 \(\xi \in (a, b)\),由于 \(f|_{(a, b)}\) 可导 \(\Rightarrow\) \(f'(\xi) = 0\)

四、\(\text{Lagrange}\) 定理

\(f(x)\)\([a, b]\) 上连续,在 \((a, b)\) 上可导,则 \(\exists \xi \in (a, b)\) 使得

\[f'(\xi) = {f(b) - f(a) \over b - a} \]

证明:令 \(g(x) = f(x) - f(a) - {f(b) - f(a) \over b - a}(x - a)\)

\(g(a) = g(b) = 0,g'(x) = f'(x) - {f(b) - f(a) \over b - a}\)

根据 \(\text{Rolle}\) 定理,\(\exists \xi \in (a, b), g'(\xi) = 0 \Rightarrow f'(\xi) = {f(b) - f(a) \over b - a}\),证毕

六、\(\text{Cauchy}\) 中值定理

\(f(x), g(x)\)\([a, b]\) 上连续,\((a, b)\) 上可导,且 \(g'(x) \neq 0\),则 \(\exists \xi \in (a, b)\),使得

\[{f'(\xi) \over g'(\xi)} = {f(b) - f(a) \over g(b) - g(a)} \]

证明与 \(\text{Lagrange}\) 定理证明类似

定理

\(f(x)\)\([a, b]\) 上连续,在 \((a, b)\) 上可导,若 \(f'(x)\)\(x_0\) 的左(右)极限存在,则 \(f(x)\)\(x_0\) 的左(或右)导数满足

\(f'_{\pm}(x_0) = f'(x_0 \pm 0) \Rightarrow f'(x_0 - 0) = f'(x_0 + 0) = f'(x_0) \Rightarrow\) \(f'(x)\)\(x_0\) 处连续,\(f'(x)\) 的间断点一定无左右极限

结论:导函数不存在第一类间断点

七、\(\text{Darbsux}\) 定理(导函数的介值性)

\(f|_{[a, b]}\) 可导,则对于介于 \(f'(a), f'(b)\) 之间的任意值 \(\lambda\),一定 \(\exists \xi \in [a, b]\),使得 \(f'(\xi) = \lambda\)

证明:

不妨设 \(f'(a) < f'(b)\),并考虑 \(f'(a) < 0 < f'(b)\)

\(\Rightarrow \lim\limits_{x \to a^+}{f(x) - f(a) \over x - a} = f'(a) < 0\)

\(\Rightarrow \exists \delta_1\) 使得当 \(x \in (a, a + \delta_1)\)\({f(x) - f(a) \over x - a} < 0 \Rightarrow f(x) < f(a)\)

即当 \(x \in (a, a + \delta_1)\) 时,\(f(x) < f(a)\),同理当 \(x \in (b - \delta_2, b)\) 时,\(f(x) < f(b)\)

\(\Rightarrow \min f(x)\) 一定在 \((a, b)\) 内部,即 \(\exists \xi \in (a, b)\) 使得 \(f'(\xi) = 0\)

对于 \(f'(a) < \lambda < f'(b)\),令 \(g(x) = f(x) - \lambda x \Rightarrow g'(a) < 0 < g'(b)\)

\(\Rightarrow \exists \xi \in (a, b)\),使 \(g'(\xi) = 0 \Rightarrow f'(\xi) = \lambda\),证毕

八、\(\text{L'Hospital}\) 法则

结论:

\[\lim\limits_{x \to x_0}{f'(x) \over g'(x)} = l \Rightarrow \lim\limits_{x \to x_0}{f(x) \over g(x)} = l \]

证明:

\(\frac 0 0\) 型:

因为 \(\lim\limits_{x \to x_0}f(x) = 0, \lim\limits_{x \to x_0}g(x) = 0\)

不妨取 \(f(x_0) = 0, g(x_0) = 0 \Rightarrow f(x), g(x)\)\(x_0\) 处连续

\[{f(x) \over g(x)} = {f(x) - f(x_0) \over g(x) - g(x_0)} = {f'(\xi) \over g'(\xi)} \]

其中 \(|\xi - x_0| < |x - x_0|\),所以当 \(x \to x_0, \xi \to x_0\)

\[\Rightarrow \lim\limits_{x \to x_0}{f(x) \over g(x)} = \lim\limits_{\xi \to x_0}{f'(\xi) \over g'(\xi)} = l \]

\(\frac \infty \infty\) 型:

类似地取 \(\xi, x_0 < \xi < x_0 + \delta\)

\[{f(x) \over g(x)} = {f(x) - f(\xi) \over g(x) - g(\xi)} - {f(x) - f(\xi) \over g(x) - g(\xi)}{g(\xi) \over g(x)} + {f(\xi) \over g(x)} \]

\(x \to x_0, \xi \to x_0\)

\[{f(x) \over g(x)} \to {f'(x_0) \over g'(x_0)} \]

可以推导出高阶导和趋于无穷的情况,不再赘述。

七、单调性与凸性

一、单调性

\(f(x)\) 可导 \(\Rightarrow \begin{cases} f(x) \uparrow & \Rightarrow f'(x) \geqslant 0 \\ f(x) \downarrow & \Rightarrow f'(x) \leqslant 0\end{cases}\) ,极值点处一定有 \(f'(x) = 0\)

二、凸性

若对 \(\forall \alpha \in (0, 1)\), 有

\[f(x_1 + (1 - \alpha)(x_2 - x_1)) \leqslant f(x_1) + (1 - \alpha)(f(x_2) - f(x_1)) \]

或者写成

\[f(\alpha x_1 + (1 - \alpha)x_2) \leqslant \alpha f(x_1) + (1 - \alpha)f(x_2) \\ \]

则称 \(f(x)\) 在定义域内是凸函数

由定义可推出等价结论:

\[{f(x) - f(x_1) \over x - x_1} \leqslant {f(x_2) - f(x_1) \over x_2 - x_1} \leqslant {f(x_2) - f(x) \over x_2 - x} \]

定理:

\(f(x)\)\(I\) 上连续

  1. \(f|_I\) 可导,则 \(f(x)\)\(\iff f'(x)\uparrow\)(严格)
  2. \(f|_I\) 二阶可导,则 \(f(x)\)\(\iff f''(x) > 0\)
  1. 证明:

    \((\Rightarrow)\) 分别求端点导数即可

    \((\Leftarrow)\) 拉格朗日中值

  2. 证明同理可得

三、拐点

\(f(x)\) 凸性改变的点叫拐点

定理:

\(x_0\) 的邻域内,满足在 \(x_0\) 左边 \(f'(x) \uparrow\),右边 \(f(x) \downarrow\),则 \(x_0\) 是拐点

四、曲率

\[\alpha = \arctan{\text dy \over \text dx} = \arctan y' \Rightarrow {\text d\alpha \over \text dx} = {|y''| \over 1 + y'^2}\\ \kappa = {\text d \alpha \over \text d s} = {\text d \alpha \over \text d x} / {\text d s \over \text d x} = {|y''| \over 1 + y'^2} / \sqrt{1 + y'^2} = {|y''| \over (1 + y')^{\frac 3 2}} \]

用参数方程表示,设 \(x = \varphi(t), y = \psi(t), y = f(x)\)

\[\begin{aligned} \Rightarrow f' = {\psi' \over \varphi'}, f'' = {\psi''\varphi' - \varphi''\psi' \over \varphi'^3} \\ \Rightarrow \kappa = {\psi''\varphi' - \varphi''\psi' \over (\psi'^2 + \varphi'^2)^{\frac 3 2}} \end{aligned} \]

八、\(\text{Taylor}\) 展开

一、\(\text{Taylor}\) 公式

\(f(x)\)\(x_0\) 附近 \(n\) 阶可导

\(\text{Peano}\) 余项公式:

\[f(x) = f(x_0) + {f'(x_0) \over 1!}(x - x_0) + \cdots {f^{(n)}(x_0) \over n!}(x - x_0)^n + o((x - x_0)^n) \]

特别地当 \(x_0 = 0\) 时有:

\(\text{Maclaurim}\) 公式:

\[f(x) = f(0) + {f'(0) \over 1!}(x) + \cdots {f^{(n)}(0) \over n!}(x)^n + o(x^n), x \to 0 \]

待定系数法+多次 \(\text{L'Hospital}\) 法则可以求得各系数的值,满足余项是 \(n\) 次项的高阶无穷小

二、余项的估计

\(f(x)\)\(x_0\) 的领域内有 \(n + 1\) 阶导数,\(T_n(x)\)\(f(x)\)\(x_0\) 处的 \(n\)\(\text{Taylor}\) 多项式

则在 \(x\)\(x_0\) 之间 \(\exists \xi\),使得

\[f(x) = T_n(x) +{f^{(n + 1)} \over (n + 1)!}(x - x_0)^{n + 1} \]

证明:

设辅助函数

\[g(t) = f(t) - T_n(t) - {f(x) - T_n(x) \over (x - x_0)^{n+ 1}}(t - x_0)^{n + 1}, t \in I \]

\(g(t)\)\(n + 1\) 阶导数,且满足

\[g(x_0) = g(x) = 0, g^{(k)}(x_0) = f^{(k)}(x_0) - T_n^{k}(x_0) = 0, k = 1, 2, \cdots, n \]

重复运用微分中值定理可推得

\[\exists \xi \in (x_0, x), g^{(n + 1)}(\xi) = f^{(n + 1)}(\xi) - (n + 1)!{f(x) - T_n(x)\over(x - x_0)^{n + 1}} \]

\[f(x) = T_n(x) + {f^{(n + 1)}(\xi) \over (n + 1)!}(x - x_0)^{n + 1} \]

并令

\[R_n(x) = {f^{(n + 1)}(\xi) \over (n + 1)!}(x - x_0)^{n + 1} \]

\(R_n(x)\) 称作 \(\text{Lagrange}\) 余项

推论:

  1. \(f(x)\)\(n + 1\) 阶导数有界,\(|f^{(n + 1)}(x)| \leqslant M \Rightarrow \Rightarrow |R_n(x)| \leqslant {M \over (n + 1)!}|x - x_0|^{n + 1}\)

  2. \(f(x)\)\(I\)\(n + 1\) 阶可导,\(x_0\)\(f(x)\)\(r(r < n)\) 重零点当且仅当存在函数 \(g(x)\)

    \[f(x) = (x - x_0)^rg(x), g(x_0) \neq 0 \]

三、初等函数的展开式

下面给出部分初等函数具有 \(\text{Lagrange}\) 余项的 \(\text{Maclaurim}\) 公式

\[\begin{aligned} & e^x = 1 + x + {x^2 \over 2!} + \cdots + {x^n \over n!} + {e^{\theta x} \over (n + 1)!}x^{n+1}, & -\infty < x < +\infty, \\ & \ln(1+x) = x - {x^2 \over 2!} + \cdots + (-1)^{n - 1}{x^n \over n} + {(-1)^nx^{n+1}\over(n+1)(1+\theta x)^{n+1}}, & x > -1, \\ & \sin x = x - {x^3 \over 3!} + \cdots + (-1)^{m - 1}{x^{2m - 1} \over (2m - 1)!} + {(-1)^mx^{2m+1} \over (2m+1)!}\cos \theta x, & -\infty < x < +\infty, \\ & \cos x = 1 - {x^2 \over 2!} + \cdots + (-1)^{m - 1}{x^{2m - 2} \over (2m - 2)!} + {(-1)^mx^{2m} \over (2m)!}\cos \theta x, & -\infty < x < +\infty, \\ & (1+x)^\alpha = 1 + \alpha x + {\alpha(\alpha - 1) \over 2!}x^2 + \cdots + {\alpha^{\underline{n}} \over n!}x^n + {\alpha^{\underline{n + 1}} \over (n + 1)!}x^{n + 1}(1 + \theta x)^{\alpha - n - 1}, & x > -1 \end{aligned} \]

结论:初等函数在定义域内任意一点可以展开成任意阶的 \(\text{Taylor}\) 多项式

\(\text{Taylor}\) 展开的一些用途:

  1. 可以用 \(\text{Taylor}\) 展开方便地求函数的高阶导
  2. 化为多项式方便计算极限
posted @ 2021-09-15 23:19  Sangber  阅读(385)  评论(1编辑  收藏  举报